Suite.(clin d'oeil à MMu)

Bonsoir,
<BR>
<BR>Une suite fabriquée utilisant des outils déjà vus ..
<BR>Suite réelle définie par : <P></P><DIV ALIGN="CENTER" CLASS="mathdisplay"><IMG WIDTH="259" HEIGHT="48" ALIGN="MIDDLE" BORDER="0" SRC="http://www.les-mathematiques.net/phorum/2006/12/8/103635/cv/img1.png&quot; ALT="$\displaystyle u_0>0,\ u_1>0,\quad u_{n+2}=\frac{u_{n+1}+u_n} { u_{n+1}^2+u_n^2} $"></DIV><P></P>B...t connait peut-être, sinon il va certainement cogiter ::))
<BR>
<BR>Oump.
«1

Réponses

  • Well... Let me see.
  • Yo

    On peut remarquer (en faisant le changement de variable $v_n = 1/u_n$ et en bidouillant quelques inégalités) que limsup = 1/liminf. Ca vous rappelle rien ?
  • Au premier coup d'oeil, j'ai pensé à regarder $v_n=\frac{u_{n+1}}{u_n}$. Mais difficile d'en déduire directement quelque chose.
  • Re

    Finalement j'ai seulement limsup >= 1/liminf. L'autre sens n'est pas immédiat a priori ! ...
  • Si limite l il y a , alors l=0 ou l=1 .

    oump: tu inventes ces suites la nuit ,ou alors, tu les proposais auparavant à tes étudiants?
    merci
  • Enfin, quand je disais difficile d'en déduire quelque chose, je parlais du comportement asymtptotique. Pour les limites, c'est bon.
  • Enfin, quand je disais difficile d'en déduire quelque chose, je parlais du comportement asymptotique. Pour les limites, c'est bon.
  • 0 n'est pas limite car il est répulsif (ainsi que $+\infty$). En effet:

    $u_{n+2}=\left( \frac{u_{n}+u_{n+1}}{\sqrt{u_{n}^{2}+u_{n+1}^{2}}} \times \frac{1}{\sqrt{u_{n}^{2}+u_{n+1}^{2}}}$

    Le premier facteur est dans $[1;\sqrt{2}]$. Quand $u_{n}$ et $u_{n+1}$ sont dans un voisinage de zéro, $u_{n+2}$ est grand.
  • Vraiment du beau monde sur ce forum !


    [Ne pas se laisser impressionner par les pseudo choisis :
    jean-louis Ovaert alias lucien Chambadal
    qui ont la même adresse mail : lili1 et accessoirement la même adresse IP. AD]
  • tiens tiens! salut Jean louis

    ( pour info Oumpapah=Claude Becker )

    pour Bs , apparemment elle est de moi ; mais va savoir si elle ne traine pas qq part!

    Pour Guimauve

    on a bien existence de m>0 et M>0 avec pour tout n
    m<=u(n)<=M
    et ensuite on a bien liminfu et limsup u inverses l'une de l'autre

    la cle: pour x et y strictement positifs on a betement
    :)) 1/max(x,y)<= (x+y)/(x²+y²) <= 1/min(x,y)

    et ensuite on passe de l'ensemble des quasimajorants de u à l'ensemble de ses quasiminorants par t->1/t

    on a fait un bout du chemin , à vous de poursuivre..

    A+ Oump.
  • Bonsoir,

    Une suite fabriquée utilisant des outils déjà vus ..
    Suite réelle définie par : $$ u_0>0,\ u_1>0,\quad u_{n+2}=\frac{u_{n+1}+u_n} { u_{n+1}^2+u_n^2} $$ B...t connait peut-être, sinon il va certainement cogiter ::))

    Oump.



    $$u_{n+2}= \frac{u_{n}+u_{n+1}}{\sqrt{u_{n}^{2}+u_{n+1}^{2}}} \times \frac{1}{\sqrt{u_{n}^{2}+u_{n+1}^{2}}}$$
  • bonjour,

    je rédige une preuve avec mes outils:

    1) soient $u_{0}$ et $u_{1}$ les valeurs initiales strictement positives.
    D'après ce qui précède, la suite de terme général $u_{n}$ est bornée
    entre $\min(u_{0},u_{1},\frac{1}{u_{0}},\frac{1}{u_{1}}$

    2) De cette suite bornée, on en extrait une sous suite convergente

    3) La sous suite convergente a nécessairement pour limite 1

    4) Pour tout $\varepsilon > 0 \quad \exists n \quad |u_{n}-1|< \min(1+\varepsilon,\frac{1}{1+varepsilon}$
    (on prend un intervalle $I_{\varepsilon}$ autour de 1 invariant par
    passage à l'inverse)
    Tous les termes de la suite au delà de l'entier $n$ restent dans cet intervalle.
    La suite converge donc vers 1.
  • errata:
    lire:
    1)
    la suite est bornée entre $\min \left( u_{0},u_{1},\frac{1}{u_{0}},\frac{1}{u_{1}} \right)$ et $\max \left( u_{0},u_{1},\frac{1}{u_{0}},\frac{1}{u_{1}} \right)$

    4) Pour tout $\varepsilon \quad > 0 \exists n \quad |u_{n}-1|< \min \left( \varepsilon, \frac{\varepsilon}{1+\varepsilon} \right)$
  • Jean-Louis : n'est-ce pas plutôt $u_n \in ]\frac{1}{1+ \varepsilon}, 1+\varepsilon[$ ?
  • zut, j'ai écrit des bêtises. Il faudrait deux termes consécutifs dans l'intervalle...
  • moi, je n'ai jamais vu le vrai Ovaert écrire des bêtises..
    donc je conseille au faux Ovaert de ne pas ridiculiser le nom du vrai...
  • je suis vraiment confus. j'ai le niveau inférieur au capes. Avec toute mes excuses.
  • ça n'a rien à voir avec le niveau,
    ne jamais faire des bétises n'existe pas,
    que ce soit Oavert, ou n'importe qui
    ne jamais écrire des bétises, c'est possible, surtout si on n'écrit pas,
    mais pour ceux qui veulent faire un geste, et aider un peu, il peuvent écrire des bétises...
    c'est un compromis, l'aventure c'est l'aventure, on ne fait pas d'omelette sans casser des oeufs,
    c'est un forum, avec ses avantages et ses inconvénients.
  • Bonjour,
    merci Alain de nous avoir informé sur cette usurpation d'identité via le pseudo ! On frole le faux et usage de faux ...
    Si d'autres signatures de mathématiciens célèbres toujours en activité s'avèrent être également de grossières copies de l'original, il est fort appréciable de nous en informer, comme tu l'as si bien fait..Merci.
    Revenons aux maths.
  • Après toutes ces suites il me semble qu'on peut tenter d'unifier et de généraliser. Je propose pour l'ordre $2$ :

    Soit $F$ une fonction continue de ${\R^+}^2$ dans ${\R^+}$ telle que l'équation $F(x,x)=x$ admette une solution dans $\R^+$ alors la suite définie par $u_0=x>0,\,u_1=y>0$ et $u_{n+2}=F(u_{n+1},u_n)$ converge vers cette solution.
  • On peut rajouter que $F$ est symétrique.
  • une sorte de théorème du point fixe ?
  • Oui, en quelque sorte un th du point fixe entre 2 espaces de dimensions différentes. J'ai fait moult essais avec des fonctions $F$ différentes. Ca marche tout le temps. Peut on prouver que, d'une certaine façon, $F$ est "contractante"?
  • B....t : Ca ne marche pas en posant F(x,y)=x+y. On doit avoir une condition suffisante si les sup des dérivées partielles sont majorés par 1/2 en valeur absolue.
  • Pour Egorrof, $F$ n'a pas à être forcément symétrique. Essaie $F(x,y)=(x^3+y^2)/(x+y^3)$ qui donne des suites convergent vers $1$.
  • Oui Guimauve, il faudrait peut être simplement préciser que l'image de $F$ est un intervalle borné de $R^+$.
  • OK, merci B...t. J'avais dit ça parce que c'était le cas pour les suites proposées par Oump. N'empêche que j'ai peine à croire que ça marche pour toute fonction $F$ en supposant seulement la continuité.
  • Re

    F(x,y)=0 si x>=1
    F(x,y)=1-x si x<1

    C'est bien continu et l'image est bornée, mais ça ne fonctionne pas (par exemple en prenant u0=0, u1=1/2).
  • bonjour,

    comment démontrer que la suite proposée converge vers 1.
    ma démo,question (4) ne marche pas.

    signé: le faux Ovaert qui bat humblement sa coulpe.
  • J'ai lancé l'idée comme ça sans trop réfléchir aux conditions. Il s'avère que ça marche pour beaucoup d'autres fonctions. L'exemple de Guimauve me fait penser que l'ensemble de Mandelbrot rentre un peu dans ce champ et qu'il doit être difficile de trouver des conditions trop générales.
  • Bonsoir ,
    <BR>
    <BR>Pour B...t
    <BR>Je réfléchis à ce que tu proposes ; il me semble que les hypothèses sont trop larges ; mais il y a du grain à moudre.
    <BR>
    <BR>Cela dit personne n'a encore donné une preuve simple pour ma suite.
    <BR>Tu a sûrement une idée, vu ce que tu dis, j'aimerais la comparer à la mienne...
    <BR>
    <BR>Et tant que j'y suis une autre suite intéressante : <P></P><DIV ALIGN="CENTER" CLASS="mathdisplay"><IMG WIDTH="259" HEIGHT="50" ALIGN="MIDDLE" BORDER="0" SRC="http://www.les-mathematiques.net/phorum/2006/12/9/103730/cv/img1.png&quot; ALT="$\displaystyle u_0>0,\ u_1 >0,\quad u_{n+2} = \frac{2u_n u_{n+1}}{u_{n+1}^2+u_n^2} $"></DIV><P></P>prouver la convergence vers 1 bien sûr (dans ce cas <SPAN CLASS="MATH"><IMG WIDTH="82" HEIGHT="32" ALIGN="MIDDLE" BORDER="0" SRC="http://www.les-mathematiques.net/phorum/2006/12/9/103730/cv/img2.png&quot; ALT="$ F(x,x)=1$"></SPAN> pour tout <SPAN CLASS="MATH"><IMG WIDTH="12" HEIGHT="13" ALIGN="BOTTOM" BORDER="0" SRC="http://www.les-mathematiques.net/phorum/2006/12/9/103730/cv/img3.png&quot; ALT="$ x$"></SPAN>...)
    <BR>
    <BR>Son traitement utilise des idées tournant toujours autour du même thème.
    <BR>Il y a un moment que ces suites récurrentes me tracassent...
    <BR>
    <BR>Et encore un thème à exploiter : <P></P><DIV ALIGN="CENTER" CLASS="mathdisplay"><IMG WIDTH="259" HEIGHT="57" ALIGN="MIDDLE" BORDER="0" SRC="http://www.les-mathematiques.net/phorum/2006/12/9/103730/cv/img4.png&quot; ALT="$\displaystyle u_0 >0,\ u_1 >0,\quad u_{n+2} = \frac{u_{n+1}^2 +u_n^2}{u_{n+1}+u_n} $"></DIV><P></P> (ici <SPAN CLASS="MATH"><IMG WIDTH="83" HEIGHT="32" ALIGN="MIDDLE" BORDER="0" SRC="http://www.les-mathematiques.net/phorum/2006/12/9/103730/cv/img5.png&quot; ALT="$ F(x,x)=x$"></SPAN> pour tout <SPAN CLASS="MATH"><IMG WIDTH="12" HEIGHT="13" ALIGN="BOTTOM" BORDER="0" SRC="http://www.les-mathematiques.net/phorum/2006/12/9/103730/cv/img3.png&quot; ALT="$ x$"></SPAN>...). Cette suite converge pour tout choix de <SPAN CLASS="MATH"><IMG WIDTH="19" HEIGHT="29" ALIGN="MIDDLE" BORDER="0" SRC="http://www.les-mathematiques.net/phorum/2006/12/9/103730/cv/img6.png&quot; ALT="$ u_0$"></SPAN> et <SPAN CLASS="MATH"><IMG WIDTH="19" HEIGHT="29" ALIGN="MIDDLE" BORDER="0" SRC="http://www.les-mathematiques.net/phorum/2006/12/9/103730/cv/img7.png&quot; ALT="$ u_1$"></SPAN>.
    <BR>En posant <SPAN CLASS="MATH"><IMG WIDTH="50" HEIGHT="29" ALIGN="MIDDLE" BORDER="0" SRC="http://www.les-mathematiques.net/phorum/2006/12/9/103730/cv/img8.png&quot; ALT="$ u_0=x$"></SPAN> et <SPAN CLASS="MATH"><IMG WIDTH="49" HEIGHT="29" ALIGN="MIDDLE" BORDER="0" SRC="http://www.les-mathematiques.net/phorum/2006/12/9/103730/cv/img9.png&quot; ALT="$ u_1=y$"></SPAN> on a la limite <SPAN CLASS="MATH"><IMG WIDTH="52" HEIGHT="32" ALIGN="MIDDLE" BORDER="0" SRC="http://www.les-mathematiques.net/phorum/2006/12/9/103730/cv/img10.png&quot; ALT="$ F(x,y)$"></SPAN> fonction de <SPAN CLASS="MATH"><IMG WIDTH="40" HEIGHT="32" ALIGN="MIDDLE" BORDER="0" SRC="http://www.les-mathematiques.net/phorum/2006/12/9/103730/cv/img11.png&quot; ALT="$ (x,y)$"></SPAN>
    <BR><SPAN CLASS="MATH"><IMG WIDTH="15" HEIGHT="14" ALIGN="BOTTOM" BORDER="0" SRC="http://www.les-mathematiques.net/phorum/2006/12/9/103730/cv/img12.png&quot; ALT="$ F$"></SPAN> doit être continue (cela fait penser aux équa diff quand on étudie les solutions, fonction de conditions initiales)
    <BR>J'ai idée que tu as une large bibliographie sur les suites, j'imagine que tu vas y jeter un oeil ?
    <BR>
    <BR>J'oubliais : dans les suites que j'étudie il y a toujours de l'homogénéité pour <SPAN CLASS="MATH"><IMG WIDTH="15" HEIGHT="14" ALIGN="BOTTOM" BORDER="0" SRC="http://www.les-mathematiques.net/phorum/2006/12/9/103730/cv/img12.png&quot; ALT="$ F$"></SPAN>... à voir de plus près .
    <BR>
    <BR>A+ ( ah qu'il est doux de ne plus avoir de contraintes professionnelles pour se livrer sans retenue à ces futiles travaux ::))
    <BR>
    <BR>Oump.<BR>
    <BR><BR>[En LaTeX, c'est plus clair :) AD]
  • Bonsoir ,

    Pour B...t
    Je réfléchis à ce que tu proposes ; il me semble que les hypothèses sont trop larges ; mais il y a du grain à moudre.

    Cela dit personne n'a encore donné une preuve simple pour ma suite.
    Tu a sûrement une idée, vu ce que tu dis, j'aimerais la comparer à la mienne...

    Et tant que j'y suis une autre suite interessante : $$ u_0>0,\ u_1 >0,\quad u_{n+2} = \frac{2u_n u_{n+1}}{u_{n+1}^2+u_n^2} $$ prouver la convergence vers 1 bien sûr (dans ce cas $F(x,x)=1$ pour tout $x$...)

    Son traitement utilise des idées tournant toujours autour du même thème.
    Il y a un moment que ces suites récurrentes me tracassent...

    Et encore un thème à exploiter : $$ u_0 >0,\ u_1 >0,\quad u_{n+2} = \frac{u_{n+1}^2 +u_n^2}{u_{n+1}+u_n} $$ (ici $F(x,x)=x$ pour tout $x$...). Cette suite converge pour tout choix de $u_0$ et $u_1$.
    En posant $u_0=x$ et $u_1=y$ on a la limite $F(x,y)$ fonction de $(x,y)$
    $F$ doit être continue (cela fait penser aux équa diff quand on étudie les solutions, fonction de conditions initiales)
    J'ai idée que tu as une large bibliographie sur les suites, j'imagine que tu vas y jeter un oeil ?

    J'oubliais : dans les suites que j'étudie il y a toujours de l'homogénéité pour $F$... à voir de plus près .

    A+ ( ah qu'il est doux de ne plus avoir de contraintes professionnelles pour se livrer sans retenue à ces futiles travaux ::))

    Oump.
  • Et la bonne vieille récurrence
    $$u_{n+2} = \dfrac{u°n^2 + u_{n+1}^2}{2}$$
    où $F(x,y) = \dfrac{x^2 + y^2}{2}$ ??
  • Re

    rien ne parait simple

    1.soit la suite definie par u(0) et u(1) >0 et

    u(n+2)=( 2u(n+1)²+u(n)²)/(u(n+1)²+2u(n)²)
    au premier abord
    convergence vers 1

    2.soit la suite définie par v(0) et v(1)>0 et

    v(n+2)=( u(n+1)² +2u(n)²)/(2u(n+1)²+u(n)²)
    " à la machine " il apparait " 2 valeurs d'adhérence 1.722083805 et 0.5806918321
    limites des suites d'indice pairs et d'indices impairs

    et il traine la meme fonction de deux variables à echange pres de x et y
    ( d'ou importance ..on n'a pas les memes resultats avec
    f(x,y) et g(x,y)=f(y,x) )

    pour gb : je vais regarder ta suite

    j'ai mis les pieds dans un drole de truc..

    A+
    Oump.
    ( et une preuve de la convergence vers 1 de ma suite initiale?
    comme georges me dit secher , je vais regarder ma preuve de pres et vous en ferai part le moment venu!)
  • "v(n+2)=( u(n+1)² +2u(n)²)/(2u(n+1)²+u(n)²)
    " à la machine " il apparait " 2 valeurs d'adhérence 1.722083805 et 0.5806918321..."

    Valeurs intéressantes car ce sont appremment les racines positives de :

    X^4 - X^3 - X^2 - X + 1=0

    J'avoue ne pas saisir. Il y a effectivement des cas très particuliers!

    Pour v(n+2)=( u(n+1)² +A*u(n)²)/(A*u(n+1)²+u(n)²) avec A>1 j'observe que v(n) converge suivant la partié de n systématiquement vers les 2 racines réelles positives de :

    X^4 - (A-1)*(X^3+ X^2+ X) + 1=0
  • Alors apparut la constante d'Oumpapah! Je pars sur la dernière de Oump que je paramètre avec $A>0$:

    $v_{n+2}=\frac{Av_{n+1}^2+v_{n}^2}{v_{n+1}^2+Av_{n}^2}$

    alors je subodore l'existence d'une valeur $\omega>1$ telle que :

    $v_n$ converge vers $1$ si $0\omega$,
    $v_{2n}$ et $v_{2n+1}$ convergent chacune vers les $2$ racines réelles de $x^4-(A-1)(x^3+x^2+x)+1=0$.

    Je pense que $\omega=5/3$ car pour $A=5/3$ on a $(x-1)^2$ qui divise $x^4-(A-1)(x^3+x^2+x)+1=0$.
  • concernant la suite initiale:
    $u_{n+2}=\frac{u_{n}+u_{n+1}}{u_{n}^{2}+u_{n+1}^{2}}$
    on peut regarder l'écart $f(x,y)-1$ en fonction des écrats $x-1$ et $y-1$:
    $x-1;y-1;f(x,y)-1$ où $f(x,y)=\frac{x+y}{x^{2}+y^{2} \qquad f(1,1)=1$
    $f(x,y)-1=f(x,y)-f(1,1)=f(x,y)-f(1,y)+f(1,y)-f(1,1)$
    $f(x,y)=(x-1)f_{1}^{'}(1,y)+(y-1)f_{2}^{'}(1,1)+$ termes du second ordre
    $f(x,y)=(x-1)\left(\frac{-2+(y-1)^{2}}{{(1+y^{2})}^{2}}\right)-\frac{1}{2}(y-1)$
    +termes du second ordre

    Il semble que pour x et y proches de 1, f n'est pas contractante.
    La suite n'admettrait pas 1 pour limite.
  • Bonsoir,

    Pour confirmer l'observation de B.....t :

    Si $u_{2k} \to l$ et $u_{2k+1} \to l'$, alors on les deux égalités $P(l,l') = Q(l,l') =0$, où

    $P(x,y) = (Ay^2 + x^2)x - y^2 - Ax^2$
    $Q(x,y) = (Ax^2 + y^2)y - x^2 - Ay^2 = P(y,x).$

    On élimine la variable $y$ à l'aide de la théorie des résultants :

    $l$ est racine de $Res_x(P,Q) = (a-1)^2(a+1)^2x^4(x-1)[x^4+(1-A)(x^3+x^2+x) + 1]$.

    Si $l = 0$, alors $l' = 0$, ce qui est exclu.
    Si $l = 1$ alors $l' = 1$.

    Sinon, $l$ est bien racine de $x^4+(1-A)(x^3+x^2+x) + 1$, et de même, $l'$ l'est aussi.

    Cordialement,

    Ritchie
  • L'étude de la suite "homographico-quadratique" d'ordre 2 :
    <P></P><DIV ALIGN="CENTER" CLASS="mathdisplay"><IMG WIDTH="149" HEIGHT="57" ALIGN="MIDDLE" BORDER="0" SRC="http://www.les-mathematiques.net/phorum/2006/12/9/103769/cv/img1.png&quot; ALT="$\displaystyle v_{n+2}=\frac{a\,v_{n+1}^2+b\,v_{n}^2}{c\,v_{n+1}^2+d\,v_{n}^2} $"></DIV><P></P>
    est assez drôle.<BR>
  • Merci Ritchie, la constante d'Oumpapah s'explique donc bien. Egoroff avait raison, il faudrait tenir compte des solutions de $F(x,y)=x$ et $F(x,y)=y$ et alors la suite converge vers une limite unique ou double.
  • concernant la suite initiale:
    $u_{n+2}=\frac{u_{n}+u_{n+1}}{u_{n}^{2}+u_{n+1}^{2}}$
    la seule limite possible est 1.
    $u_{n+2}-1=\left( \frac{- u_{n}}{u_{n}^{2}+u_{n+1}^{2}} \right)
    (u_{n}-1)+\left( \frac{- u_{n+1}}{u_{n}^{2}+u_{n+1}^{2}} \right)
    (u_{n+1}-1)$
    quand $u_{n}$ et $u_{n+1}$ sont proches de 1,
    la suite des écarts de terme général $v_{n}=u_{n}-1$ vérifient grosso modo la relation:
    $v_{n+2}=-\frac{1}{2}(v_{n}+v_{n+1})$
    cette dernière relation admet des solutions périodiques car le discriminant de son équation caractéristique est négatif.
    Ceçi indique, à mon avis, que la suite de terme général $u_{n}$ n'admet en général pas 1 pour limite.
  • L'étude de la suite "homographico-quadratique" d'ordre 2 : $$ v_{n+2}=\frac{a\,v_{n+1}^2+b\,v_{n}^2}{c\,v_{n+1}^2+d\,v_{n}^2} $$ est assez drôle.
  • Bonjour,

    A mon réveil (tardif) je vois que ça usine sec..

    Reste, Benoit à prouver ce que tu dis sur "ma constante" ::))
    Il y a bien des choses à faire concernant ces suites...

    Pour Lulu ; la suite converge bien vers 1; j'ai donné une preuve dans le fil
    <http://www.les-mathematiques.net/phorum/read.php?f=2&i=339915&t=339915&gt;
    sur les triangles diophantiens en réponse à Georges.
    La méthode employée parait féconde dans pas mal de situations
    ( par ex pour la suite que j'ai aussi proposée
    u(n+2)= 2u(n)u(n+1)/(u(n)²+u(n+1)²)
    A+ Oump.
  • Re

    le fil a pour intitulé exact "aire d'un triangle"
  • Bonjour Claude,

    Je reproduis ici ta preuve avec, entre parenthèses, mon doute :

    Avec f(x,y)=(x+y)/(x^2+y^2) ( x et y >0)
    on a 1/max(x,y) <= f(x,y) <= 1/min(x,y)

    (Donc, si x<=y, on a 1/y<=f(x,y) <= 1/x
    d'où, en posant a =max(x, 1/x, y, 1/y)>=1, on a 1/a<=f(x,y), 1/f(x,y) <= a)

    on en deduit quasi immédiatement que si m est un quasi minorant de u
    alors 1/m en est un quasi majorant, et de meme si M est un quasimajorant de u alors 1/M en est un quasiminorant

    (on en déduit une suite (a(n)) décroissante dans [1, + infini[ tq. pour tout n>=2, on a :
    1/a(n)<=u(n)<=a(n),
    donc en posant b=inf a(n)>=1, on a :
    1/b<=liminf u(n)<=limsup u(n)<=b
    mais cela ne prouve pas ton assertion)

    dans un premier temps on a u(n) compris entre disons 1/a et a avec 1<a
    dans un deuxieme temps on limsup et liminf finies et inverses l'une de l'autre .

    (c'est là qu'est mon doute)

    Ensuite , à grand renfort de suites extraites on a le resultat suivant cle de tout:( c'est la que j'ai mis du temps à comprendre la dialectique de MMu!)

    si b est une valeur d'adherence il existe c,d,e valeurs d'adhérence telles que
    b=(c+d)/(c^2+d^2) et c= (d+e)/(d^2+e^2)

    soit T la limsup et 1/T la liminf

    on a T=(c+d)/(c^2+d^2) avec c et d >=1/T
    on a c^2+d^2 -c/T-d/T =0= c(c-1/T)+ d(d-1/T)=0
    d'ou c=d=1/T et d'un..
    on a alors 1/T=c=(d+e)/(d^2+e^2)
    soit dT+eT -d^2 -e^2 =0= d(T-d) +e(T-e) =0 d'ou e=d=T
    et on se souvient que d=1/T d'ou T^2=1 soit T=1 !!!

    avec une technique similaire j'ai montré la convergence vers 1 de la suite definie par u(0) et u(1) positifs et

    u(n+2) =2u(n+1)u(n)/(u(n+1)^2+u(n)^2)

    je n'avais rien fait d'aussi malin depuis un bon bout de temps.

    (Oui, très joli, si on a bien : T la limsup et 1/T la liminf).

    Amitiés,
    Georges
  • Je précise :

    (Donc, si x<=y, on a 1/y<=f(x,y) <= 1/x
    d'où, en posant a(x,y)=a =max(x, 1/x, y, 1/y)>=1, on a 1/a<=f(x,y), 1/f(x,y) <= a)

    on en deduit quasi immédiatement que si m est un quasi minorant de u
    alors 1/m en est un quasi majorant, et de meme si M est un quasimajorant de u alors 1/M en est un quasiminorant

    (on en déduit une suite (a(n)=a(u(n-1),u(n-2))) décroissante dans [1, + infini[ tq. pour tout n>=2, on a
    1/a(n)<=u(n)<=a(n), donc en posant b=inf a(n)>=1, on a
    1/b<=liminf u(n)<=limsup u(n)<=b

    et limsup u(n)=b ou liminf u(n)=1/b

    mais cela ne prouve pas ton assertion)
  • Voici un exemple moins simple mais qui montre qu'il y a certainement matière à unifier. Soit :

    $F(x,y)={\sin(x+y)}^2\frac{2x+y}{x^2+2y^2}$

    alors si on part de valeurs initiales quelconques $>0$ la suite $v_{n+2}=F(v_{n+1},v_n)$ converge modulo $4$ vers une des limites :

    $l_1=1.8285515262184147776796408...$
    $l_2=0.1230524828679686166350070...$
    $l_3=0.2667821771594160442319681...$
    $l_4=0.9347171151871354582711350...$
  • Re salut Georges

    es tu d'accord que pour une suite réelle (disons bornée pour simplifier,sinon travailler dans la droite achevée) , la liminf (resp limsup)est la borne sup (resp inf)de l'ensemble des quasiminorants (resp quasimajorants)?
    c'est une des caracterisations ensemblistes que je prefere
    (mais apparemment pas classique , en france tout au moins car il me semble que d'est un point de vue abordé dans des ouvrages anglosaxons
    Rankin par ex (si ma mémoire est bonne))

    ( en notant Qn "la queue de suite" ensemble des u(k) pour k>=n

    soit a(n) l'inf de Qn et b(n) le sup de Qn on a l'intersection des segments
    [a(n) b(n)] egale à [limunf limsup]

    et si la suite est de cauchy ce segment est reduit à un point d'ou convergence , c'est comme ça que je prouvais le critere de cauchy en sup
    sans parler d'ailleurs de limsup ou inf..

    cela dit si on passe des quasimajorants aux quasi minorants par t->1/t
    on a bien
    liminf et limsup inverses l'une de l'autre non?

    A+
    PS pour B..t il me semble que tu as interverti n et n+1 dans notre suite ou tu intoduis A et de plus le produit des 2 lilites est evidemment 1quand elle oscille..a suivre

    Oump.
  • "PS pour B..t il me semble que tu as interverti n et n+1 dans notre suite ou tu intoduis A et de plus le produit des 2 lilites est evidemment 1quand elle oscille..a suivre"

    Exact, sorry : il faut alors inverser et comprendre 1/A. Le produit des racines réelles du polynôme résultant est égal au terme constant qui ici vaut 1.
  • Claude,
    je suis d'accord que : pour une suite réelle (disons bornée pour simplifier,sinon travailler dans la droite achevée) , la liminf (resp limsup)est la borne sup (resp inf)de l'ensemble des quasiminorants (resp quasimajorants)?

    Puis tu dis :
    cela dit si on passe des quasimajorants aux quasi minorants par t->1/t
    on a bien liminf et limsup inverses l'une de l'autre non?

    Cela est clair avec ce si.

    Mais comment prouves-tu que l'on passe des quasimajorants aux quasi minorants par t->1/t pour ta suite?

    Prends la suite constante x(n)=2 et vérifie, avec mes notations,
    a= a(x(0),a(x(1)))=2 et 1/a<=x(2), 1/x(2)<=a
    d'où une suite a(n)=a(x(n-1),x(n-2))=2, décroissante dans [1, + infini[ tq. pour tout n>=2, on a
    1/a(n)<=x(n)<=a(n), donc en posant b=inf a(n)=2, on a
    1/2< liminf x(n)=limsup x(n)=2<=b

    et on ne passe pas des quasimajorants aux quasi minorants par t->1/t

    @+
Connectez-vous ou Inscrivez-vous pour répondre.